Ball og skråplan

Her kan brukere av forum utfordre hverandre med morsomme oppgaver og nøtter man ønsker å dele med andre. Dette er altså ikke et sted for desperate skrik om hjelp, de kan man poste i de andre forumene, men et sted for problemløsing på tvers av trinn og fag.

Moderatorer: Vektormannen, espen180, Aleks855, Solar Plexsus, Gustav, Nebuchadnezzar, Janhaa

Svar
daofeishi
Tyrann
Tyrann
Innlegg: 1486
Registrert: 13/06-2006 02:00
Sted: Cambridge, Massachusetts, USA

For de som prøver lykken i Statene universitetsmessig til neste år, nærmer SAT seg med stormskritt (O Glede...) Da kan det vel kanskje passe seg å prøve seg på fysikkproblemer?

En baseball er gitt en starthastighet [tex]v_0[/tex] med en vinkel [tex]\phi[/tex] i forhold til et skråplan, som selv holder en vinkel [tex]\theta[/tex] over horisontalen.

a) Finn lengden, målt langs skråplanet, til punktet ballen treffer på skråplanet.

b) Hva slags vinkel [tex]\phi[/tex] vil gi ballen maksimum rekkevidde langs skråplanet?
Charlatan
Guru
Guru
Innlegg: 2499
Registrert: 25/02-2007 17:19

Vi har vinkelen [tex]\phi = \theta \ , \ \theta \in [0,\frac{\pi}{2}][/tex]
og starthastigheten [tex]v_0 \ , \ v_0 \ > \ 0[/tex]
Vi regner ikke med luftmotstand.
Vi lager en parameterframstilling for ballens starthastighet:

[tex]\vec{r}=[v_0\cos{\theta},v_0\sin{\theta}][/tex]

Vi integrerer for å finne posisjonsvektoren for ballen.

[tex]\vec{r}(t)= [v_0\cos{\theta}t,v_0\sin{\theta}t][/tex]

Må ikke glemme gravitasjonskraften (Vi setter gravitasjonskraften lik g, den har et måltall på ca [tex]9.81[/tex]): [tex]\vec{g}=[0,-g][/tex]
Siden dette er en akselerasjonsvektor integrerer vi en gang for å finne fartsvektoren, og enda en gang for å finne posisjonsvektoren:

[tex]\vec{g}(t)=[0,-\frac{g}{2}t^2][/tex]

Siden ballen blir påvirket av denne kraften adderer vi posisjonsvektoren til gravitasjonskraften til posisjonsvektoren til ballen:

[tex]vec{r}(t)+ \vec{g}(t) = [v_0\cos{\theta}t,v_0\sin{\theta}t -\frac{g}{2}t^2][/tex]

Vi kaller denne vektoren [tex]\vec{s}(t)[/tex]

Vi vil vite når ballen er på linje med horisontalunderlaget, altså når den vertikale komponenten er lik 0:

[tex]v_0\sin{\theta}t -\frac{g}{2}t^2 = 0 \\ t(v_0\sin{\theta} -\frac{g}{2}t)=0 \\ t=0 \ \vee \ (v_0\sin{\theta} -\frac{g}{2}t)=0 \\ \\ v_0\sin{\theta} = \frac{g}{2}t \\ t=\frac{2v_0\sin{\theta}}{g}[/tex]

Tiden 0 ser vi bort ifra siden dette tidspunktet er starttidspunktet hvor ballen selvfølgelig er ved underlaget. Det andre tidspunktet altså tidspunktet for impact.

Hvor langt den er kommet da kan vi se ved å finne ut hvor langt langs den horisontale komponenten posisjonen er ved dette tidspunktet: (Vi setter inn for t)

[tex]x=v_0\cos{\theta} \cdot \frac{2v_0\sin{\theta}}{g} = \frac{\sin{2\theta} \cdot v_o^2}{g}[/tex]

Dette er altså den horisontale lengden ballen har fra startpunktet på det horisontale underlaget. Denne lengden hadde variert hvis vinkelen [tex]\theta [/tex] hadde variert. Vi vil finne ut når denne lengden hadde vært størst, altså når faktoren [tex]\sin{2\theta}[/tex] er størst mulig: Det er den når
[tex]\sin{2\theta} = 1 \\ 2\theta = \frac{\pi}{2} + 2n\cdot \pi \ , \ n\in \mathbb{Z} \ , \ \theta \in [0,\frac{\pi}{2}][/tex]
Dette medfører at [tex]\theta = \frac{\pi}{4}[/tex]
Som vi kan gjøre om fra radianer til grader som er [tex]\theta = 45^\circ[/tex]

[tex]45 ^\circ [/tex]er altså den vinkelen som skal til for at ballen skal komme seg lengst unna.
daofeishi
Tyrann
Tyrann
Innlegg: 1486
Registrert: 13/06-2006 02:00
Sted: Cambridge, Massachusetts, USA

Jeg tror du har misforstått problemet, Jarle. Situasjonen ser slik ut:

Bilde

Endring: Ellers veldig flott arbeid :)
Sist redigert av daofeishi den 20/10-2007 14:06, redigert 1 gang totalt.
Charlatan
Guru
Guru
Innlegg: 2499
Registrert: 25/02-2007 17:19

Ånei, skal prøve igjen
arildno
Abel
Abel
Innlegg: 684
Registrert: 17/03-2007 17:19

Hvor i all verden har du det fra at phi og theta er like, Jarle??

Skråplanet har tangensialvektor [tex]\vec{t}=\cos\theta\vec{i}-\sin\theta\vec{j}[/tex], og oppad rettet normalvektor [tex]\vec{n}=\sin\theta\vec{i}+\cos\theta\vec{j}[/tex]

Vi har at utgangshastigheten danner vinkel phi med skråplan, dvs:
[tex]\vec{v}_{0}=v_{0}(\cos\phi\vec{t}+\sin\phi\vec{n})=v_{0}(\cos(\phi-\theta)\vec{i}+\sin(\phi-\theta)\vec{j})[/tex]

Vi har at [tex]\vec{j}=\cos\theta\vec{n}-\sin\theta\vec{t}[/tex]

Dermed er normal-komponenten fra tyngden:
[tex]-mg\vec{j}\cdot\vec{n}=-mg\cos\theta[/tex]

Normal-komponenten av utgangshastighet er [tex]v_{0}\sin\phi[/tex], og for å finne t-verdi for når kula kommer ned igjen, kan vi løse:
[tex]0=t*(v_{0}\sin\phi-\frac{gt\cos\theta}{2})\to{t}=\frac{2v_{0}}{g}\frac{\sin\phi}{\cos\theta}[/tex]

OBS!
Glemte tyngde-komponenten langs skråplanet nå..
arildno
Abel
Abel
Innlegg: 684
Registrert: 17/03-2007 17:19

Etter miseren i sted, som jeg har slettet, se vi at tangensialkomponenten av tyngdekraften er [tex]-mg\vec{j}\cdot\vec{t}=mg\sin\theta[/tex]

Dermed har vi at rekkevidden er gitt ved:
[tex]R=t*(v_{0}\cos\phi+\frac{g\sin\theta*t}{2})=\frac{2v^{2}_{0}\sin\phi}{g\cos\theta}(\cos\phi+\sin\phi\tan\theta)=\frac{v^{2}_{0}}{g\cos\theta}(\sin(2\phi)+tan\theta(1-cos(2\phi)))[/tex]
Hvorpå vi har at:
[tex]\frac{dR}{d\phi}=0[/tex]
impliserer [tex]\tan(2\phi)=-\cot(\theta)[/tex]
Charlatan
Guru
Guru
Innlegg: 2499
Registrert: 25/02-2007 17:19

Vi lager linja for det horisontale underlaget (Siden ballen íkke farer til sidene kan vi gjøre dette):

[tex]\vec{p}(s)=[s\cos{\theta},s\sin{\theta}][/tex]

Vi lager så framstillingen for ballens posisjon plusss gravitasjonsvektoren:

[tex]\vec{r}(t)=v_0[\cos{(\theta +\phi)}t,\sin{(\theta+\phi)}t -\frac{gt^2}{2}][/tex]

Vi vil vite hvor disse kurvene skjærer hverandre:

[tex]s\cos{\theta}=v_0\cos{(\theta +\phi)}t[/tex]
[tex]s=v_0\frac{\cos{(\theta +\phi)}}{\cos{\theta}}t[/tex]
Setter det inn for den vertikale komponenten:

[tex]s\sin{\theta}=v_0\sin{(\theta+\phi)}t -\frac{gt^2}{2}[/tex]

[tex]v_0\frac{\cos{(\theta +\phi)}}{\cos{\theta}}t \cdot \sin{\theta} = v_0\sin{(\theta+\phi)}t -\frac{gt^2}{2}[/tex]

Løser andregraslikningen og får:

[tex]t=\frac{2v_0}{g} ( sin{(\theta+\phi)-\cos{(\theta+\phi)} )[/tex]

Setter det inn for vektoren:
[tex]\vec{r}(t) = [v_0\cos{(\theta +\phi)}(\frac{2v_0}{g} ( sin{(\theta+\phi)-\cos{(\theta+\phi)} )) \ , \ v_0 \sin{(\theta+\phi)}( \frac{2v_0}{g} ( sin{(\theta+\phi)-\cos{(\theta+\phi)} ) ) -v_0 ( sin{(\theta+\phi)-\cos{(\theta+\phi)} )^2}] [/tex]

(puh)

Lengden av denne er:

Nei jeg gidder ikke mer, det må være en enklere metode. Dessuten er det stor sjans for å gjøre feil med så lange uttrykk. Hvis jeg kunne implisitt derivasjon hadde jeg kanskje ikke behøvd å ekspandere uttrykkene!
Fremgangsmåten var iallefall å regne ut lengden av vektoren, for så å derivere vektorfunksjonen med hensyn på variabelen [tex]\phi[/tex]
Tror ikke metoden er så effektiv!
arildno
Abel
Abel
Innlegg: 684
Registrert: 17/03-2007 17:19

Jeg får rette opp noen fortegnskludder, og få et resultat som lettere kobles til daofeishis tegning etterhvert..
Magnus
Guru
Guru
Innlegg: 2286
Registrert: 01/11-2004 23:26
Sted: Trondheim

Jeg finner at

[tex]l = \frac{sin(2\phi + 2\theta) - 2\tan(\theta)\cdot\cos^2(\theta + \phi)}{\cos\theta}\cdot \frac{v^2}{g}[/tex]

Skal man finne maks av dette deriverer vi alt med hensyn på phi (de andre er konstanter), noe som er heller kjedelig vil jeg tro. Er vel en mer elegant måte någen stans.

edit: Virker som om den går mot maksimum når summen av de to vinklene blir 45grader.

edit 2: Denne oppgaven er jo egentlig bare rett fram..
Sist redigert av Magnus den 20/10-2007 15:29, redigert 2 ganger totalt.
arildno
Abel
Abel
Innlegg: 684
Registrert: 17/03-2007 17:19

Vi har:

Skråplanet har tangensialvektor [tex]\vec{t}=\cos\theta\vec{i}+\sin\theta\vec{j}[/tex], og oppad rettet normalvektor [tex]\vec{n}=-\sin\theta\vec{i}+\cos\theta\vec{j}[/tex]

Vi har at utgangshastigheten danner vinkel phi med skråplan, dvs:
[tex]\vec{v}_{0}=v_{0}(\cos\phi\vec{t}+\sin\phi\vec{n})=v_{0}(\cos(\phi+\theta)\vec{i}+\sin(\phi+\theta)\vec{j})[/tex]

Vi har at [tex]\vec{j}=\cos\theta\vec{n}+\sin\theta\vec{t}[/tex]

Dermed er normal-komponenten fra tyngden:
[tex]-mg\vec{j}\cdot\vec{n}=-mg\cos\theta[/tex]

Normal-komponenten av utgangshastighet er [tex]v_{0}\sin\phi[/tex], og for å finne t-verdi for når kula kommer ned igjen, kan vi løse:
[tex]0=t*(v_{0}\sin\phi-\frac{gt\cos\theta}{2})\to{t}=\frac{2v_{0}}{g}\frac{\sin\phi}{\cos\theta}[/tex]

Dernest har vi at tangensialkomponenten av tyngdekraften er [tex]-mg\vec{j}\cdot\vec{t}=-mg\sin\theta[/tex]

Dermed har vi at rekkevidden er gitt ved:
[tex]R=t*(v_{0}\cos\phi-\frac{g\sin\theta*t}{2})=\frac{2v^{2}_{0}\sin\phi}{g\cos\theta}(\cos\phi-\sin\phi\tan\theta)=\frac{v_{0}^{2}}{g\cos\theta}(\sin(2\phi)+\tan\theta\cos(2\phi)-\tan\theta)[/tex]

Dermed får vi ved derivasjon at kravet på phi kan skrives som:
[tex]\tan(2\phi)=\cot\theta[/tex]
Sist redigert av arildno den 20/10-2007 15:24, redigert 2 ganger totalt.
Charlatan
Guru
Guru
Innlegg: 2499
Registrert: 25/02-2007 17:19

(O Glede...)
Tenker du på å begynne på universitet i statene, daofeishi? :)
arildno
Abel
Abel
Innlegg: 684
Registrert: 17/03-2007 17:19

Kravet kan videre omformes til:
[tex]sin(2\phi)\sin(\theta)=\cos(2\phi)\cos(\theta),\to\cos(2\phi+\theta)=0\to{2}\phi+\theta=\frac{\pi}{2}[/tex]

Dermed har vi enklest:
[tex]\phi=\frac{\pi}{4}-\frac{\theta}{2}, 0\leq\theta\leq\frac{\pi}{2}[/tex]
Maksimal rekkevide blir dermed lett beregnet til:
[tex]R=\frac{v_{0}^{2}}{g\cos^{2}\theta}(1-\sin\theta)=\frac{v_{0}^{2}}{g(1+\sin\theta)}, 0\leq\theta<\frac{\pi}{2}[/tex]

Hvis skråplanet skråner nedover istedet for oppover når vi beveger oss i positiv horisontalretning (slik jeg satte opp først), vil maksimal rekkevidde R gis ved å skifte ut plusstegn med minustegn i formelen.
arildno
Abel
Abel
Innlegg: 684
Registrert: 17/03-2007 17:19

En litt mer utfordrende oppgave er den følgende:

Du står i bunnen av en parabel, hvor høyden h som funksjon av horisontalvariabelen x er gitt som [tex]h(x)=\alpha{x}^{2},\alpha>0[/tex]
Gitt utgangsfart V0, finn vinkelen som maksimerer rekkevidden R, uttrykt ved[tex]v_{0},g,\alpha[/tex]
steamu
Noether
Noether
Innlegg: 30
Registrert: 07/06-2007 20:32
Sted: Stavanger

Bilde
Med forbehold om trykkfeil...
mrcreosote
Guru
Guru
Innlegg: 1995
Registrert: 10/10-2006 20:58

Jeg får maksimal rekkevidde til [tex]\frac{v^2\cos\theta}{g(1+\sin\theta)}[/tex].
arildno skrev:En litt mer utfordrende oppgave er den følgende:

Du står i bunnen av en parabel, hvor høyden h som funksjon av horisontalvariabelen x er gitt som [tex]h(x)=\alpha{x}^{2},\alpha>0[/tex]
Gitt utgangsfart V0, finn vinkelen som maksimerer rekkevidden R, uttrykt ved[tex]v_{0},g,\alpha[/tex]
Orker ikke skrive inn utregninga, så hvis noen kan være enig/uenig i svaret, er det fint. Hvis vi legger det inn i et koordinatsystem med lengdeenheter på aksene så alpha får benevning (l.e.)^{-1}, blir optimal vinkel [tex]\cos\theta = \sqrt{\frac g{2\alpha v^2+2g}}[/tex] og tilhørende maksimal lengde [tex]\frac{v^2}{\sqrt{g(g+2\alpha v^2)}}[/tex].
Svar